JEE Questions for Physics Electrostatics Ii Quiz 9 - MCQExams.com

Assertion : The coulomb force is the dominating force in the universe.
Reason : The coulomb force is weaker than the gravitational force.
  • If both assertion and reason are true but reason is not the correct explanation of the assertion
  • If both assertion and reason are true but reason is not the correct explanation of the assertion.
  • If assertion is true but reason is false.
  • If the assertion and reason both are false.
  • If assertion is false but reason is true

Physics-Electrostatics II-73714.png
  • If assertion is false but reason is true.
  • If both assertion and reason are true but reason is not the correct explanation of the assertion.
  • If both assertion and reason are true but reason is not the correct explanation of the assertion.
  • If assertion is true but reason is false.
  • If the assertion and reason both are false.
Assertion : A metallic shield in form of a hollow shell may be built to block an electric field.
Reason : In a hollow spherical shield, the electric field inside it is zero at every point.
  • If both assertion and reason are true but reason is not the correct explanation of the assertion.
  • If both assertion and reason are true but reason is not the correct explanation of the assertion.
  • If assertion is true but reason is false.
  • If the assertion and reason both are false.
  • If assertion is false but reason is true.
Assertion : Electrons move away from a low potential to high potential region.
Reason : Because electrons have negative charge
  • If both assertion and reason are true but reason is not the correct explanation of the assertion.
  • If both assertion and reason are true but reason is not the correct explanation of the assertion.
  • If assertion is true but reason is false.
  • If the assertion and reason both are false.
  • If assertion is false but reason is true
Assertion : If the distance between parallel plates of a capacitor is halved and dielectric constant is made three times, then the capacitance becomes 6 times.
Reason : Capacity of the capacitor does not depend upon the nature of the material.
  • If both assertion and reason are true but reason is not the correct explanation of the assertion.
  • If both assertion and reason are true but reason is not the correct explanation of the assertion.
  • If assertion is true but reason is false.
  • If the assertion and reason both are false.
  • If assertion is false but reason is true
Assertion : A parallel plate capacitor is connected across battery through a key. A dielectric slab of constant K is introduced between the plates. The energy which is stored becomes K times.
Reason : The surface density of charge on the plate remains constant or unchanged.
  • If both assertion and reason are true but reason is not the correct explanation of the assertion.
  • If both assertion and reason are true but reason is not the correct explanation of the assertion.
  • If assertion is true but reason is false
  • If the assertion and reason both are false.
  • If assertion is false but reason is true
Assertion : Electric lines of force cross each other.
Reason : Electric field at a point superimpose to give one resultant electric field.
  • If both assertion and reason are true but reason is not the correct explanation of the assertion.
  • If both assertion and reason are true but reason is not the correct explanation of the assertion.
  • If assertion is true but reason is false.
  • If the assertion and reason both are false.
  • If assertion is false but reason is true
Assertion : If a proton and an electron are placed in the same uniform electric field. They experience different acceleration.
Reason : Electric force on a test charge is independent of its mass.
  • If both assertion and reason are true but reason is not the correct explanation of the assertion.
  • If both assertion and reason are true but reason is not the correct explanation of the assertion
  • If assertion is true but reason is false.
  • If the assertion and reason both are false.
  • If assertion is false but reason is true
Assertion : Dielectric breakdown occurs under the influence of an intense light beam.
Reason : Electromagnetic radiations exert pressure.
  • If both assertion and reason are true but reason is not the correct explanation of the assertion.
  • If both assertion and reason are true but reason is not the correct explanation of the assertion.
  • If assertion is true but reason is false.
  • If the assertion and reason both are false.
  • If assertion is false but reason is true
Assertion : When charges are shared between any two bodies, no charge is really lost, but some loss of energy does occur.
Reason : Some energy disappears in the form of heat, sparking etc.
  • If both assertion and reason are true but reason is not the correct explanation of the assertion.
  • If both assertion and reason are true but reason is not the correct explanation of the assertion.
  • If assertion is true but reason is false.
  • If the assertion and reason both are false.
  • If assertion is false but reason is true
A simple pendulum has a length l and the mass of the bob is m . The bob is given a charge q coulomb. The pendulum is suspended between the vertical plates of a charged parallel plate capacitor. If E is the electric field strength between the plates, the time period of the pendulum is

  • Physics-Electrostatics II-73715.png
  • 2)
    Physics-Electrostatics II-73716.png

  • Physics-Electrostatics II-73717.png

  • Physics-Electrostatics II-73718.png
A fully charged capacitor has a capacitance C. It is discharged through a small coil of resistance wire embedded in a thermally insulated block of specific heat capacity s and mass m . If the temperature of the block is raised by ∆T, the potential difference V across the capacitance is

  • Physics-Electrostatics II-73720.png
  • 2)
    Physics-Electrostatics II-73721.png

  • Physics-Electrostatics II-73722.png

  • Physics-Electrostatics II-73723.png
Three capacitors C1, C2 and C3 are connected as shown in the figure to a battery of V volt. If the capacitor C3 breaks down electrically, the change in total charge on the combination of capacitors is
Physics-Electrostatics II-73725.png

  • Physics-Electrostatics II-73726.png
  • 2)
    Physics-Electrostatics II-73727.png

  • Physics-Electrostatics II-73728.png

  • Physics-Electrostatics II-73729.png
0:0:1


Answered Not Answered Not Visited Correct : 0 Incorrect : 0

Practice Physics Quiz Questions and Answers